Langkau ke kandungan utama
Microsoft
|
Math Solver
Selesaikan
Latihan
Bermain
Topik
Pra-algebra
Min
Mod
Faktor Sepunya Terbesar
Gandaan Sepunya Terkecil
Aturan Operasi
Pecahan
Pecahan Campuran
Pemfaktoran Perdana
Eksponen
Radikal
Algebra
Gabungkan Istilah Serupa
Selesaikan untuk mendapat pemboleh ubah
Faktor
Kembangkan
Menilai Pecahan
Persamaan Linear
Persamaan Kuadratik
Ketaksamaan
Sistem Persamaan
Matriks
Trigonometri
Permudahkan
Menilai
Graf
Selesaikan Persamaan
Kalkulus
Derivatif
Kamiran
Had
Input Algebra
Input Trigonometri
Input Kalkulus
Input Matriks
Selesaikan
Latihan
Bermain
Topik
Pra-algebra
Min
Mod
Faktor Sepunya Terbesar
Gandaan Sepunya Terkecil
Aturan Operasi
Pecahan
Pecahan Campuran
Pemfaktoran Perdana
Eksponen
Radikal
Algebra
Gabungkan Istilah Serupa
Selesaikan untuk mendapat pemboleh ubah
Faktor
Kembangkan
Menilai Pecahan
Persamaan Linear
Persamaan Kuadratik
Ketaksamaan
Sistem Persamaan
Matriks
Trigonometri
Permudahkan
Menilai
Graf
Selesaikan Persamaan
Kalkulus
Derivatif
Kamiran
Had
Input Algebra
Input Trigonometri
Input Kalkulus
Input Matriks
Asas
algebra
trigonometri
kalkulus
statistik
matriks
Aksara
Nilaikan
\infty
Kuiz
Limits
\lim_{ x \rightarrow 0 } \frac{1}{x^2}
Masalah Sama dari Carian Web
Showing that the \lim_{x\to 0}\frac{1}{x^2} does not exist
https://math.stackexchange.com/q/1579837
Suppose that the limit exists and equals c\in\mathbb{R}. Then for e.g. \epsilon>1 some \delta>0 must exist with \left|x\right|<\delta\implies\left|\frac{1}{x^{2}}-c\right|<1. However, if we ...
Applying L'Hopital's rule to \lim\limits_{x \to 0}\frac{2}{x^2}
https://math.stackexchange.com/questions/502024/applying-lhopitals-rule-to-lim-limits-x-to-0-frac2x2
In order to use the 0/0 case of L'Hospital's rule, we require that both the numerator and the denominator tend to 0 at the appropriate point. The numerator does not tend to 0.
Is this piece-wise function continuous, and why?
https://math.stackexchange.com/questions/2411697/is-this-piece-wise-function-continuous-and-why
If we look at the behaviour as x approaches zero from the right, the function looks like this: \begin{matrix}x & f(x) = \frac{1}{x^2} \\ 1 & 1 \\ 0.1 & 100 \\ 0.01 & 10000 \\ 0.001 & 1000000 \\ 0.0001 & 100000000\end{matrix} ...
Manipulating \lim\limits_{x \to 0}{\frac{\sqrt{x+\sqrt{x}}}{x^n}}
https://math.stackexchange.com/questions/2177214/manipulating-lim-limits-x-to-0-frac-sqrtx-sqrtxxn
If \lim\limits_{x \to 0}{\frac{\sqrt{x+\sqrt{x}}}{x^n}} = c for some c\neq 0, then \lim\limits_{x \to 0}{\frac{x+\sqrt{x}}{x^{2n}}} =c^2. Now, let \sqrt{x}=t. We then wish to find n such ...
Limit of \frac{f'(x)}{g'(x)} & g'(x) \neq 0 in Hypotheses of L'Hospital's rule.
https://math.stackexchange.com/q/110408
When we write things like \lim_{x\to a}h(x) = \lim_{x\to a}H(x) we usually mean "if either limit exists, then they both do and they are equal; if either limit does not exist, then neither limit ...
How do we calculate the Right and Left Hand Limit of 1/x?
https://math.stackexchange.com/questions/762599/how-do-we-calculate-the-right-and-left-hand-limit-of-1-x
\mathbf{Definition} : \boxed{ \lim_{x \to a^+ } f(x) = \infty } means that for all \alpha > 0, there exists \delta > 0 such that if 0<x -a < \delta, then f(x) > \alpha \mathbf{Example} ...
Lagi Item
Kongsi
Salin
Disalin ke papan klip
Masalah yang serupa
\lim_{ x \rightarrow 0 } 5
\lim_{ x \rightarrow 0 } 5x
\lim_{ x \rightarrow 0 } \frac{2}{x}
\lim_{ x \rightarrow 0 } \frac{1}{x^2}
Kembali ke atas